Difference between revisions of "2016 USAJMO Problems/Problem 4"
(Created page with "== Problem == Find, with proof, the least integer <math>N</math> such that if any <math>2016</math> elements are removed from the set <math>{1, 2,...,N}</math>, one can still ...") |
|||
Line 1: | Line 1: | ||
== Problem == | == Problem == | ||
− | Find, with proof, the least integer <math>N</math> such that if any <math>2016</math> elements are removed from the set <math>{1, 2,...,N}</math>, one can still find <math>2016</math> distinct numbers among the remaining elements with sum <math>N</math>. | + | Find, with proof, the least integer <math>N</math> such that if any <math>2016</math> elements are removed from the set <math>\{1, 2,...,N\}</math>, one can still find <math>2016</math> distinct numbers among the remaining elements with sum <math>N</math>. |
== Solution == | == Solution == | ||
+ | Since any <math>2016</math> elements are removed, suppose we remove the integers from <math>1</math> to <math>2016</math>. Then the smallest possible sum of <math>2016</math> of the remaining elements is <cmath>2017+2018+\cdots + 4032 = 1008 \cdot 6049 = 6097392</cmath> | ||
+ | so clearly <math>N\ge 6097392</math>. We will show that <math>N=6097392</math> works. | ||
+ | <math>\vspace{0.2 in}</math> | ||
+ | |||
+ | <math>\{1,2\cdots 6097392\}</math> contain the integers from <math>1</math> to <math>6048</math>, so pair these numbers as follows: | ||
+ | |||
+ | <cmath>1, 6048</cmath> | ||
+ | |||
+ | <cmath>2, 6047</cmath> | ||
+ | |||
+ | <cmath>3, 6046</cmath> | ||
+ | |||
+ | <cmath>\cdots</cmath> | ||
+ | |||
+ | <cmath>3024, 3025</cmath> | ||
+ | |||
+ | When we remove any <math>2016</math> integers from the set <math>\{1,2,\cdots N\}</math>, clearly we can remove numbers from at most <math>2016</math> of the <math>3024</math> pairs above, leaving at least <math>1008</math> complete pairs. To get a sum of <math>N</math>, simply take these <math>1008</math> pairs, all of which sum to <math>6049</math>. The sum of these <math>2016</math> elements is <math>1008 \cdot 6049 = 6097392</math>, as desired. | ||
+ | |||
+ | <math>\vspace{0.2 in}</math> | ||
+ | |||
+ | We have shown that <math>N</math> must be at least <math>6097392</math>, and that this value is attainable. Therefore our answer is <math>\boxed{6097392}</math>. | ||
Latest revision as of 13:19, 22 April 2016
Problem
Find, with proof, the least integer such that if any elements are removed from the set , one can still find distinct numbers among the remaining elements with sum .
Solution
Since any elements are removed, suppose we remove the integers from to . Then the smallest possible sum of of the remaining elements is so clearly . We will show that works.
contain the integers from to , so pair these numbers as follows:
When we remove any integers from the set , clearly we can remove numbers from at most of the pairs above, leaving at least complete pairs. To get a sum of , simply take these pairs, all of which sum to . The sum of these elements is , as desired.
We have shown that must be at least , and that this value is attainable. Therefore our answer is .
The problems on this page are copyrighted by the Mathematical Association of America's American Mathematics Competitions.
See also
2016 USAJMO (Problems • Resources) | ||
Preceded by Problem 3 |
Followed by Problem 5 | |
1 • 2 • 3 • 4 • 5 • 6 | ||
All USAJMO Problems and Solutions |